Calculus 8th Edition

Published by Cengage
ISBN 10: 1285740629
ISBN 13: 978-1-28574-062-1

Chapter 11 - Infinite Sequences and Series - 11.3 The Integral Test and Estimates of Sums - 11.3 Exercises - Page 766: 12

Answer

Divergent

Work Step by Step

\[\frac{1}{5}+\frac{1}{7}+\frac{1}{9}+\frac{1}{11}+\frac{1}{13}+\cdots=\sum_{n=1}^{\infty}\frac{1}{2n+3}\] Let us assume that, \[\sum_{n=1}^{\infty}\frac{1}{2n+3}=\sum_{n=1}^{\infty}a_n\] \[\Rightarrow a_n=\frac{1}{2n+3}\] $a_n$ is positive term and montonically decreasing. So Integral test is applicable. Let\[I=\int_{1}^{\infty}\frac{1}{2x+3}dx\] \[I=\lim_{t\rightarrow\infty}\int_{1}^{t}\frac{1}{2x+3}dx\;\;\;\;\;\;\;\ldots(1)\] Let \[I_1=\int\frac{1}{2x+3}dx\] Substitute \[y=2x+3\;\;\Rightarrow\;dy=2dx\;\Rightarrow dx=\frac{1}{2}dy\] \[\Rightarrow I_1=\frac{1}{2}\int \frac{dy}{y}=\frac{1}{2}\ln |y|\] \[\Rightarrow I_1=\frac{1}{2}\ln |2x+3|\;\;\;\;\;\ldots (2)\] Using (2) in (1) \[I=\frac{1}{2}\lim_{t\rightarrow\infty}[\ln |2x+3|]_{x=1}^{t}\] \[I=\frac{1}{2}\lim_{t\rightarrow\infty}(\ln |2t+3|-\ln |5|)\] \[I=\infty\] $I$ is divergent. By integral test, $\displaystyle\sum_{n=1}^{\infty}\frac{1}{2n+3}$ is divergent. Hence, $\displaystyle{\frac{1}{5}+\frac{1}{7}+\frac{1}{9}+\frac{1}{11}+\frac{1}{13}+\cdots}$ is divergent
Update this answer!

You can help us out by revising, improving and updating this answer.

Update this answer

After you claim an answer you’ll have 24 hours to send in a draft. An editor will review the submission and either publish your submission or provide feedback.